Which type of energy is produced
by vibrations traveling in waves?
A. chemical energy
B. nuclear energy
C. sound energy

Answers

Answer 1

Answer:

C. sound energy

Explanation:

hope this helps


Related Questions

Which of the following best describes a consumer?
0. a business owner who sells clothes
0. someone who studies economics
O. a farmer who sells his crops
O. a person who buys goods and services

Answers

Answer:

a business who buys goods and services

( TRUE false )

1.)manuals are the organic matter.

2.)combine is used for sowing the seeds.

3.) the first agricultural task is sowing of seeds.

4.) many plants are first grown in Kinder Gardens and then sown in fields.

5.) earthworm are farmer friendly.

6.)substance that kills
pests are called weedicides.

7.) animals which provide us both egg and flesh are called milch animals.

8.) Grains should be dried in Sun before storage.

9.)wheat,gram,barley,potato, are kharif crops.

10.) the crops on in June /July and harvested in September/ October are called rabi crops.


plz answerzz fast​

Answers

Answer:

number 6 is wrong

the answer is pesticides

2.combines are used for harvesting or threshing.so number 2 is wrong

at least you tried

Explanation:

there is no such thing as weedicides

Answer:

1. True

2.True

3.False

4. True

5. True

6.False

7.False

8.True

9.False

10.False

An atom of unknown element Z has mass number of 39 and an atomic number of 18. How many protons and how many neutrons are in this atom? Show your work

Answers

Answer:

proton:18

neutron:39-18=21

If we removed the producers from this food web, how would it affect the balance of the ecosystem?

Answers

Answer:

In a food web, the removal of any trophic level upsets the balance within the web and can cause its eventual collapse. Because producers capture solar energy and convert it to food energy, their loss would affect every other level of the food web.

Explanation:

The removal of the producers would cause the collapse of the entire food web. Primary consumers or herbivores, which feed on producers directly, would die off. The next to be affected would be the secondary consumers or carnivores that eat the primary consumers. Higher level consumers would suffer as organisms from lower trophic levels start to die off. Decomposers would break down the bodies of dead organisms, returning their basic elements and compounds to the environment. However, even these dead organisms would run out and the entire food web would collapse.

Hope you like the answer.

A skydiver falls toward the ground at a constant velocity. Which statement best applies Newton’s laws of motion to explain the skydiver’s motion? An upward force balances the downward force of gravity on the skydiver. The skydiver’s inertia is zero, which no longer allows for acceleration. An unbalanced force pulls the skydiver toward the ground. The mass of the skydiver is so great that there is no acceleration

Answers

Answer: A) an upward force balances the downward force of gravity on the skydiver

Explanation: I got it correct on my test on edge

The statement best applies Newton’s laws of motion to explain the skydiver’s motion an upward force balances the downward force of gravity on the skydiver. Thus, option A is correct.

Which statement best applies Newton’s laws of motion?

The statement that best applies Newton’s laws of motion to explain the skydiver’s motion is that an upward force balances the downward force of gravity on the skydiver. Newton's 3rd law often applies to skydiving.

When gravity is not acting upon the skydivers they would continue moving in the direction the vehicle they jumped from was moving. If no air resistance takes place, then the skydivers would still accelerating at 9.8 m/s until they hit the ground.

The skydiver after leaving the aircraft will accelerates downwards due to the force of gravity usually as there is no air resistance acting in the upwards direction, and there is a resultant force acting downwards, the skydiver will accelerates towards the ground.

Therefore, The statement best applies Newton’s laws of motion to explain the skydiver’s motion an upward force balances the downward force of gravity on the skydiver. Thus, option A is correct.

Learn more about skydiver on:

https://brainly.com/question/29253407

#SPJ6

A physical inventory on December 31 shows 3500 units on hand. Bonita sells the units for $15 each. The company has an effective tax rate of 20%. Bonita uses the periodic inventory method. The weighted-average cost per unit is

Answers

The answer would be 10,500 be cause you multiply 3500 times 15 and you get 52500 the you multiply that by .20 and you get 10,500.

i) A 100 W and 60 W bulb are joined in series and connected to the mains. Which bulb
will glow brighter? Why?
ii) A 100 W and a 60 W bulb are joined in parallel to the mains. Which bulb will
glow brighter? Why?​

Answers

Answer:

i) 60 W

ii) 100 W

Explanation:

In each case, the bulb that dissipates the most power is the bulb that glows brighter.  Power is voltage times current (P = VI).  Using Ohm's law, we can rewrite this as P = I²R or P = V²/R.

Bulbs are rated at a certain power for a certain voltage.  P = V²/R, so the bulb with the lower resistance will have the higher power rating.  Therefore, the 100 W bulb has a lower resistance than the 60 W bulb.

i) They are in series, so they have the same current.  P = I²R, so the bulb with the higher resistance will glow brighter.  That's the 60 W bulb.

ii) They are in parallel, so they have the same voltage.  P = V²/R, so the bulb with the lower resistance will glow brighter.  That's the 100 W bulb.

What form of energy does a block of chocolate have?

Answers

Answer:

Chemical Energy

Explanation:

If a body is moving in the horizontal axis with a velocity Vx= 6m/s and in the vertical axis Vy=8m/s What is the angle Theta above the horizontal axis of the velocity of the body V? A. Theta = 45.0 degrees B.Theta = 63.3 degrees C. Theta = 53.1 degrees D. Theta = 43.7 degrees

Answers

Answer: C

Explanation: It's a lot of math.

The option that gives the correct value for the angle theta above the horizontal axis of the velocity of the body, v is option C.

C. 53.1°

The given parameters are;

The velocity of the body relative to the horizontal axis, vₓ = 6 m/s

The velocity of the body relative to the vertical axis, [tex]v_y[/tex] = 8 m/s

Required;

The measure of the angle theta above above the horizontal axis of the velocity of the body, v

Solution:

The velocity of the body can be represented vectorially as follows;

v = 6·i + 8·j

The vectoral form gives the legs of the right triangle that has the magnitude and direction as the hypotenuse side

Therefore;

[tex]\theta = arctan \left(\dfrac{8}{6} \right) \approx \underline{53.1^{\circ}}[/tex]

Learn more about the vectors here:

https://brainly.com/question/16854117

https://brainly.com/question/16958684

https://brainly.com/question/17097378

state three effects of malnutrition in farm animals​

Answers

Answer:

Cows,Pigs,Chicken. They are always in farm

Question is on the picture. Answers: A. 0.1 J/g*C B. 0.2 J/g*C C. 0.4 J/g*C D. 4 J/g*C

Answers

Answer:

B. 0.2 J/g/°C

Explanation:

The solid phase is the first segment (from 0°C to 50°C).

q = mCΔT

200 J = (20 g) C (50°C)

C = 0.2 J/g/°C

we hammer the broader side of a nail and not its sharp end to put it in wood .why ?

Answers

Explanation:

We hammer the broader side of a nail instead of its sharp end to put it in wood. It is a concept based on the pressure exerted.

The force acting per unit area of an object is called pressure exerted. It means that pressure is inversely proportional to the area of cross section i.e. area increases, pressure decreases and vice versa.

The broader side of a nail has more area as compared to the sharp end. It means that the broader side will exert less pressure on them which make it easier for us to fix it into the wood.

Explain what happens to the sound waves when a singer hits the high pitched notes during the National Anthem. Be sure to use the terms amplitude and frequency in your answer.

Answers

Answer: The frequency increases as the pitch increases, and the amplitude increases as the volume increases

Explanation:

Waves have the property of:

v = f*λ

where v is the speed of the wave (which is almost constant for soundwaves, v = 340 m/s)

f is the frequency of the wave, and λ is the wavelength.

Now, we know that when the pitch of a note increases, also does the frequency of the soundwave (so the wave oscillates faster).

Now, we also want to include the amplitude of the soundwave in this.

The amplitude is related to the volume of the soundwave (actually is related to the energy, and as higher is the energy, more "loud" is the sound).

As the high pitch part is usually "louder", we can assume that we have an amplitude increase.

Then the answer would be:

"The frequency increases as the pitch increases, and the amplitude increases as the volume increases"

A control clinic offers a program that guarantees a weight loss of up to 0.46 kg in one week. Express the weight loss in a ratio of milligrams per second.

Answers

Answer:

0.76 mg/s

Explanation:

0.46 kg/week × (1 week / 7 days) × (1 day / 24 hrs) × (1 hr / 3600 s) × (1000 g/kg) × (1000 mg/g) = 0.76 mg/s

If the boy on the bicycle in the preceding problem accelerates from rest to a speed of 10.0 m/s in 10.0 s, the angular acceleration of the tires is:_______

Answers

Answer:

The angular acceleration is 3.33 rad/s²

Explanation:

Here is the preceding problem as well as the given problem:

A boy rides his bicycle 2.00 km. The wheels have radius 30.0 cm. What is the total angle the tires rotate through during his trip?

21.

If the boy on the bicycle in the preceding problem accelerates from rest to a speed of 10.0 m/s in 10.0 s, what is the angular acceleration of the tires?

Explanation:

To determine the angular acceleration, we will first find the linear acceleration

From one of the equations of kinematics for linear motion

v = u + at

Where v is the final speed

u is the initial speed

a is the linear acceleration

and t is time

From the question,  the boy starts from rest, hence,

u = 0.0 m/s

v = 10.0 m/s

t = 10.0 s

a = ??

From, v = u + at

a = [tex]\frac{ v - u}{t}[/tex]

a = [tex]\frac{10 - 0}{10}[/tex]

a = 1 m/s²

Now, for the angular acceleration, [tex]\alpha[/tex]

The relationship between linear acceleration, a and angular acceleration, [tex]\alpha[/tex] is

a = [tex]\alpha[/tex]r

where r is radius

From the previous question, r = 30.0cm = 0.3m

From the equation, a = [tex]\alpha[/tex]r

[tex]\alpha[/tex] = a / r

[tex]\alpha[/tex] = 1 / 0.3

[tex]\alpha[/tex] = 3.33 rad/s²

PLEASE ASAP! I SENT A PHOTO CHECK AND TELL!

Answers

Answer:

Answer B is the correct answer: "Motion of one projectile as seen from the other is a straight line."

Explanation:

Let's write the equations of motion for each projectile, using that projectile [tex]a[/tex] is launched with velocity [tex]a[/tex] which has components associated with the angle of launching, given in x and y coordinates as: [tex]a_x\,\,and\,\,a_y[/tex].

Similarly, assume that projectile b is launched with velocity [tex]b[/tex] with components due to the launching angle = [tex]b_x\,\,and \,\,b_y[/tex]

then the equations of motion for the two projectiles launched at the same time (t) from the same spot (position that we assume to be at the origin of coordinates to simplify formulas) are:

[tex]x_a=a_x\,t\\y_a= a_y\,t-\frac{1}{2} g\,t^2\\and\\x_b=b_x\,t\\y_b= b_y\,t-\frac{1}{2} g\,t^2[/tex]

therefore, from the frame of reference of projectile "b", the x and y position of projectile "[tex]a[/tex]" would be:

[tex]x_{a\,b}= x_a-x_b= a_x\,t-b_x\,t=(a_x-b_x)\,t[/tex]  which is linear in "t"

[tex]y_{a\,\,b}=y_a-y_b= a_y\,t-\frac{1}{2} g\,t^2-\left[ b_y\,t-\frac{1}{2} g\,t^2\right]=(a_y-b_y)\,t[/tex] which is also linear in t.

Therefore the motion of one projectile with reference to the other is a straight line (answer B)

Notice as well that this two projectiles cannot collide because they have been launched together, and supposedly at different speeds and angles. The only way that they can share the same x-coordinate and the same y-coordinate at the same time "t" is if their velocity components are equal, which is not what we are told.

[tex]x_a=x_b\\a_x\,t= b_x\,t\\and\\y_a= y_b\\a_y\,t-\frac{1}{2} g\,t^2= b_y\,t-\frac{1}{2} g\,t^2\\a_y\,t=b_y\,t\\a_y=b_y[/tex]

Dr. Bishop wants to study how small children respond to scary stimuli. She will explain the procedure to parents and ask them to sign a permission form. She will have 3-5 year old kids individually come into her lab
Each child will be given a teddy bear to cuddle with. She will then stand a couple of feet behind the child as he or she plays and make a very loud scary sound. She will observe and measure the child's responses. The
child will come back in a few days and be given the teddy bear again, she will measure any reactions the child has to the bear.
What might an ethics review committee decide about the proposal?
The proposal is approved, as it meets the ethical guidelines defined by the APA
O The proposal is approved, because there is no deception
The proposal is denied, because participants wil ikely suffer psychological harm.
O The proposal is denied, because participants are not allowed to leave the study
The proposal is denied, because it does not meet confidentiality standards

Answers

Answer:

The correct option is;

The proposal is denied, because participants will likely suffer psychological harm

Explanation:

Among the recommendations in the report on ethical research with children, it is suggested that the following should be observed by researchers;

1) Seeking consent should be a continuous process

2) Contemplate payment forms other than money to allow more discretion on child safety

3) Have two way communication with the child

4) Ask for assent of the child participating in the research

The above APA suggestions where not met by the researcher in the study.

Answer:

The proposal is denied, because participants will likely suffer psychological harm.

Explanation:

This proposal would cause psychological harm to its participents.

I also took the test and that was the correct answer.

What two factors determine how much potential energy an object has?

Answers

Answer:

The mass of the object and its height in the gravitational field of the Earth.

Explanation:

If we are talking about gravitational potential energy which is defined as:

[tex]U=m\,*\,g\,*\,h[/tex]

being "m" the object's mass, "g" the acceleration due to gravity, and "h" the height at which the object is located relative to the conventionally picked level for zero of potential energy.

As long as the value of "g" is constant, the only two variables that determine the gravitational potential energy are the mass (m) of the object and its relative height (h).

Answer:

The objects weight and height above Earth's surface

Explanation:

K12 :)

To see if your results are reasonable, you can compare the final velocity of the stone as it falls down unwinding the wire from the pulley, to the velocity the stone would have if falling the same distance while unconnected to the pulley. What is the velocity of an untethered stone after falling 0.337 m from rest

Answers

Answer:

The velocity of the stone is 2.57 m/s.

Explanation:

Given that

Height = 0.337 m

We need to calculate the velocity of the stone

Using equation of motion

[tex]v^2-u^2=2gh[/tex]

Where, v = velocity of stone

u = initial velocity

g = acceleration due to gravity

h = height

Put the value into the formula

[tex]v^2-0=2\times9.8\times 0.337[/tex]

[tex]v=\sqrt{2\times9.8\times0.337}[/tex]

[tex]v=2.57\ m/s[/tex]

Hence, The velocity of the stone is 2.57 m/s.

numeria
problems
lifted
Joy
1
لوووو
G. Solue the
following
A load
800 Nis
effort
If
the load is
laced at
a distance
Lo cun from
achat will be the
effort distance ?
Solher load ( l = 8oon
Effort
(e) = 200N
the fularum,​

Answers

Answer:

[tex] \boxed{40 \: cm}[/tex]

Explanation:

Load ( L ) = 800 N

Effort ( E ) = 200 N

Load distance ( LD ) = 10 cm

Effort distance ( ED ) = ?

now, Let's find the effort distance:

[tex] \mathsf{L \times LD = E \times ED}[/tex]

Plug the values

[tex] \mathsf{800 \times 10 = 200 \times ED}[/tex]

Multiply the numbers

[tex] \mathsf{8000 = 200 \: ED}[/tex]

Swipe the sides of the equation

[tex] \mathsf{200 \: ED = 8000}[/tex]

Divide both sides of the equation by 200

[tex] \mathsf{ \frac{200 \: ED }{200} = \frac{8000}{200} }[/tex]

Calculate

[tex] \mathsf{ED \: = \: 40 \: cm}[/tex]

Hope I helped!

Best regards!

1. Determina el momento que produce una fuerza de 7 N tangente a una rueda de un metro de diámetro, sabiendo que el punto de aplicación es el mismo borde de dicha rueda provocando un impulso en el sentido de las agujas del reloj. Solución:

Answers

Answer:

τ= F r     into the blade

Explanation:

The moment of a force is defined by

         τ = F x r

where the bold indicates vectors

Let us write in the expression in magnitude

         τ = F r sin θ

in our case the force is tangent to the wheel therefore the angle between F and the radius is 90º, and the sin 90 = 1

       τ= F r

The direction of τ can be used by the rule of the right hand, the fingers curve in the direction of the torque when advancing from the force to the radius and the thumb points in the direction of the torque.

In this case, for a clockwise rotation, the fingers are curved in the direction and the thumb points into the blade, this is the direction of the τ.

TRASLATE

El momento de una fura es definido por

         τ = F x r

donde la negrillas indican vectores

Escribamos en ta expresión en magnitud

          τ = F r sin θ

en nuestro caso la fuerza es tangente a la rueda por lo tanto el angulo entre F y el radios es 90º, y el sin 90=1

        τ = F r

la dirección de tau la podemos  usar la regla de la mano derecha, los dedos curva en la dirección del torque al avanzar dese la fuerza al radio y el pulgar apunta en la dirección del torque.

En este caso para un giro en sentido horario los dedos se curvan ente sentido y el pulgar apunta hacia dentro de lla hoja, esta es la dirección del troque

Un pez llamado PARGO ROJO vive a grandes profundidades. Si se pesca, al salir a la superficie puede tomar el aspecto de la foto porque su vejiga natatoria aumenta de volumen al disminuir la presión. La ley que lo explica es...

Answers

Answer:

Hay diversas leyes que podemos usar acá.

Acá sabemos que la vejiga aumenta su tamaño al reducir la presión, esto tiene sentido, pues al haber menos presión, hay menos fuerza que comprime la vejiga, lo que le permite aumentar su volumen.

Acá tenemos una relación inversa de la forma: V = K/P

Una relación inversa donde la presión esta en el denominador y K es un termino que no depende ni del volumen ni de la presión.

Entonces, a medida que aumenta P, el denominador aumenta, por lo que el valor del volumen decrece.

Un ejemplo de una ecuación similar es la del gas ideal, por ejemplo, para un gas ideal dentro de un globo de volumen V para una dada presión P:

V = nRT/P

donde n es el numero de moles, R es la constante termodinámica y T es la temperatura, acá podemos ver que esta ecuación tiene la misma forma fundamental que la escrita arriba.

Gravity is affected by the ?

Answers

Two major factors, mass and distance, affect the strength of gravitational force on an object. Newton's law of gravitation describes very accurately how mass and distance affect the force of gravity.

Answer:

Mass and distance

Explanation:

The force of gravity the masses exert on each other. If one of the masses is doubled, the force of gravity between the objects is doubled. increases, the force of gravity decreases. Hence, answer is mass and distance

What is the equation for burning a fossil fuel

Answers

Answer:

Fuel + O2 → CO2 + H2O

Explanation:

The quantum state of a particle can be specified by giving a complete set of quantum numbers (n,l, ml,ms). How many different quantum states are possible if the principal quantum number is n=3?

Answers

Explanation:

We need to find the quantum state of a particle can be specified by giving a complete set of quantum numbers (n,l, ml,ms).

We have the principal quantum number, n = 3

The value of l = n-1

l = 0,1,2,3

The value of [tex]m_l[/tex]

[tex]m_l=-l\ \text{to}\ +l\\\\m_l=-3,-2,-1,0,1,2,3[/tex]

The value of [tex]m_s[/tex]

[tex]m_s=\dfrac{-1}{2},\dfrac{+1}{2}[/tex]

Hence, this is the required solution.

A man of mass 75kg stands on a weighing machine in a lift. Determine the reading of the weighing machine when the lift is moving: 1. Upwards with an acceleration of 2m/s² 2. Downwards with a constant velocity of 1.5m/s 3. Downwards with an acceleration of 2.5m/s²

Answers

Explanation:

Given that,

Mass of a man, m = 75 kg

(1) We need to find the reading of the weighing machine when the lift is moving upwards with an acceleration of 2 m/s². When the lift is moving upward, reading of the weighing machine is given by :

R = m(g+a)

R = 75 (9.8+2)

R = 885 N

(2) When the lift is moving downwards with a constant velocity of 1.5m/s. It means that the lift is at rest. So, the reading is :

R = mg

R = 75 (9.8)

R = 735 N

(3) When the lift is moving downwards with an acceleration of 2.5 m/s². In this case,

R = m(g-a)

R = 75 (9.8-2.5)

R = 547.5 N

Hence, this is the required solution.

in this system, potential and kinetic energy are ____ proportional.

Answers

Answer:

Inversely proportional

Explanation:

P.E ∞ h

K.E ∞ v

v ∞ 1/h

Therefore

K.E ∞ 1/h

Then

P.E ∞ 1/(K.E)

Answer:

Inversely

Explanation:

Astronomers were at first surprised to find complicated molecules in the interstellar medium. They thought ultra-violet light from stars would break apart such molecules. What protects the molecules we observe from being broken apart

Answers

Answer:

The dust present in the clouds.

Explanation:

The complicated composition molecules that can be found in space are generally associated with clouds of dust. The significant amount of dust in these clouds provides protection not only for these molecules, but for any body that makes up or is associated with dust clouds.

It is exactly this dust that protects the molecules against the action of ultraviolet rays.

a object 1.5cm high produces a real image 2cm high. placed at a distance of 20cm
from a concave mirror calculate: [a] the position of the image [b] focal lenght of the concave mirror

Answers

Answer:

a. 26.7 cm. b. 11.4 cm.

Explanation:

a. We know h'/h = d'/d where h' = image height = + 2 cm (since it is a real image), h = object height = + 1.5 cm, d' = image distance from mirror and d = object distance from mirror = 20 cm

So, from h'/h = d'/d

d = h'd/h

= 2 cm × 20 cm/1.5 cm

= 40/1.5 cm

=  26.67 cm

≅ 26.7 cm

The position of the image is 26.7 cm from the mirror

b. Using the mirror formula

1/d + 1/d' = 1/f where d = object distance from mirror = + 20 cm, d' = image distance from mirror = + 26.7 cm (its positive since its a real image) and f = focal length of mirror.

So, 1/d + 1/d' = 1/f

⇒ f = dd'/(d + d')

= 20 cm × 26.7 cm/(20 cm + 26.7 cm)

= 534/46.7

= 11.43 cm

≅ 11.4 cm

The focal length of the mirror is 11.4 cm

Students are going to conduct an experiment to study the effect of a net force applied to an object on the object’s motion. In each trial of the experiment, the students will apply a net force on the object. They also need to take two other measurements. What are the other quantities they should measure in each trial of the experiment?(1 point) velocity and time mass and acceleration mass and velocity acceleration and time

Answers

Answer:

A. velocity and time

Explanation:

A force can be define as an agent which has the capacity to change the state of an object. It can either increase the velocity of a body, change its direction of motion or cause a moving object to come to rest.

From Newton's second law of motion;

F = ma

where F is the force on the object, m is the mass of the object and a is the acceleration of the object. The unit of force is kgm/[tex]s^{2}[/tex] or Newtons.

a = [tex]\frac{change in velocity}{change in time}[/tex]

In the given question, apart from the mass of the object which is constant, the students should take the measurements of the velocity and time in each trial so as to calculate the required acceleration.

Answer:

mass and acceleration!

Explanation:

Because i am in class just like you and got it wrong because i didn't choose   my first answer:(

Other Questions
A chicken soup recipe calls for 13 cups of chicken stock how much is this in quarts Becky's ship is 43 miles west all the harbor.Clyde's yacht is a5 miles north from Beery. Howfar is Clyde from the Harbor? Show your work.x= Harbor25B.43 how many unique 10 digit numbers can be formed if the number 2 is in the first place and repetition is allowed? Digby's balance sheet has $99,131,000 in equity. Further, the company is expecting net income of 3,000,000 next year, and also expecting to issue $4,000,000 in new stock. If there are no dividends paid what will beDigby's book value Access controls are enforced automatically in FMS service routines that access and manipulate files and directories.a Trueb. False The Bookstall Inc. is a specialty bookstore concentrating on used books sold via the Internet. Paperbacks are $1.35 each, and hardcover books are $3.50. Of the 60 books sold last Tuesday morning, 55 were paperback and the rest were hardcover. What was the weighted mean price of a book? (Round your answer to 2 decimal places.) Compare and contrast spatial variation and spatial association Which value of x makes the following matrix equation true? Drag each tile to the correct box.Match each literary device with the reason authors use it.alliterationoxymoronsymbolismimagerymakes readers pause and think about the contradiction created with two opposingwordscreates a clear picture in the reader's mindrepresents an abstract idea with a concrete objectcalls attention to certain words and creates a rhythm with words2020 Edmentum. All rights reserved.epersuade - Google..O End of Semester TeFotoran Doud if the fixed cost for the Job Shop were changed to $305,000, what would the new break-even point in numbers of units The _____ act is where ideas get explained, the nooks and crannies of characters are explored, and the reasons for characters come into effect.It is important to keep in mind where characters start and _____.The more complications that arise out of the _________ and ____________ are what make the plot more interesting.An _____ does not need to be physical.Simple stories like "The Three Little Pigs" which have a beginning, a middle, and an ending show _______.Sometimes it is a good idea when writing a story to start at the _____ and then work backwards.The ____ act is where all the questions get answered.The _______ is how the story ends, the pay off, the third act. good is excludable if: a. it is Wi-Fi or a similar service. b. people who do not pay cannot be easily prevented from using the good. c. one person's use of the good does not reduce the ability of another person to use the same good. d. people who do not pay can be easily prevented from using the good. Find all values of x on the graph of f(x) = 2x3 + 6x2 + 7 at which there is a horizontal tangent line. a diagonal of rectangle forms a 30 degree angle with each of the longer sides of the rectangle. if the length of the shorter side is 3, what is the length of the diagonal the fastest growing major religion in the world is Which phrase best completes the sentence? Margarita: Fuimos _________________________ unas faldas, unos cinturones de marca, un vestido para Juanita y un lente nuevo para mi cmara. A. al supermercado y compramos B. a la feria artesanal y compramos C. al almacn y compramos D. al mall y compramos The value of y varies jointly with x and z. If y = 7 when z = 196 and x = 2, find the value of y when x = 3 and z = 336. I will rate you brainliest Solve for w. | w|2 Write a compound inequality like 1 < x < 3 or like x < 1 or x > 3. Use integers, proper fractions, or improper fractions in simplest form. According to the passage, what is one purpose of dance rituals in Iroquois culture? to keep people healthy and limber to ensure a good harvest to prepare for a burial ceremony to honor the right-handed twin Health-Tech Materials is a firm that manufactures medical equipment purchased by hospitals and clinics. The firm employs over 2,500 workers in its two manufacturing facilities located in Texas and Arizona. Recently, Health-Tech dismissed a number of employees for ethics violations ranging from improper Internet usage to stealing company funds. In addition, the firm has been sued by two former employees for wrongful discharge. As a result, Health-Tech's top executives are evaluating the firm's hiring process, ethics training system, reward system, and disciplinary system to determine how to promote ethics and fair treatment.Health-Tech's HR department has devised a brief ethics test for employees to use when determining if their actions match Health-Tech's code of conduct. Which of the following questions is LEAST likely to be included on the ethics test?A) What Health-Tech employees will be affected by my actions?B) Will my actions reflect negatively on the image of Health-Tech?C) Would I perform this action if I worked for a Health-Tech competitor?D) Is the action I am about to perform on behalf of Health-Tech a legal one?